Difference between revisions of "2022 AIME I Problems/Problem 13"

(Solution 1)
m (Solution 2 (similar(?) to solution 1 but reworded (not exactly for clarity)))
 
(17 intermediate revisions by 11 users not shown)
Line 5: Line 5:
 
==Solution 1==
 
==Solution 1==
  
<math>0.abcd=\frac{\overline{abcd}}{9999}</math>, <math>9999=9\times 11\times 101</math>.
+
<math>0.\overline{abcd}=\frac{abcd}{9999} = \frac{x}{y}</math>, <math>9999=9\times 11\times 101</math>.
  
Then we need to find the number of positive integers less than 10000 can meet the requirement.Suppose the number is x.
+
Then we need to find the number of positive integers <math>x</math> that (with one of more <math>y</math> such that <math>y|9999</math>) can meet the requirement <math>1 \leq {x}\cdot\frac{9999}{y} \leq 9999</math>.
  
Case 1: (9999, x)=1. Clearly x satisfies. <cmath>\varphi \left( 9999 \right) =9999\times \left( 1-\frac{1}{3} \right) \times \left( 1-\frac{1}{11} \right) \times \left( 1-\frac{1}{101} \right)=6000</cmath>
+
Make cases by factors of <math>x</math>. (A venn diagram of cases would be nice here.)
  
Case 2: 3|x but x is not a multiple of 11 or 101. Then the least value of abcd is 9x, so that <math>x\le 1111</math>, 334 values from 3 to 1110.
 
  
Case 3: 11|x but x is not a multiple of 3 or 101. Then the least value of abcd is 11x, so that <math>x\le 909</math>, 55 values from 11 to 902.
+
Case <math>A</math>:
  
Case 4: 101|x. None.
+
<math>3 \nmid x</math> and <math>11 \nmid x</math> and <math>101 \nmid x</math>, aka <math>\gcd (9999, x)=1</math>.
  
Case 5: 3, 11|x.  Then the least value of abcd is 11x, 3 values from 33 to 99.
+
Euler's totient function counts these:  
 +
<cmath>\varphi \left(3^2 \cdot 11 \cdot 101 \right) = ((3-1)\cdot 3)(11-1)(101-1)= \bf{6000}</cmath> values (but it's enough to note that it's a multiple of 1000 and thus does not contribute to the final answer)
  
To sum up, the answer is <cmath>6000+334+55+3=\boxed{392}</cmath> mod <math>1000</math>.
+
Note: You don't need to know this formula. The remaining cases essentially re-derive the same computation for other factors of <math>9999</math>. This case isn't actually different.
 +
 
 +
The remaining cases have <math>3</math> (or <math>9</math>), <math>11</math>, and/or <math>101</math> as factors of <math>abcd</math>, which cancel out part of <math>9999</math>.
 +
Note: Take care about when to use <math>3</math> vs <math>9</math>.
 +
 
 +
 
 +
Case <math>B</math>: <math>3|x</math>, but <math>11 \nmid x</math> and <math>101 \nmid x</math>.
 +
 
 +
Then <math>abcd=9x</math> to leave 3 uncancelled, and <math>x=3p</math>,
 +
so  <math>x \leq \frac{9999}{9} = 1111</math>, giving:
 +
 
 +
<math>x \in 3 \cdot \{1, \dots \left\lfloor \frac{1111}{3}\right\rfloor\}</math>,
 +
 
 +
<math>x \notin (3\cdot 11) \cdot \{1 \dots \left\lfloor \frac{1111}{3\cdot 11}\right\rfloor\}</math>,
 +
 
 +
<math>x \notin (3 \cdot 101) \cdot \{1 \dots \left\lfloor \frac{1111}{3 \cdot 101}\right\rfloor\}</math>,
 +
 
 +
for a subtotal of <math>\left\lfloor \frac{1111}{3}\right\rfloor -  (\left\lfloor\frac{1111}{3 \cdot 11}\right\rfloor + \left\lfloor\frac{1111}{3 \cdot 101}\right\rfloor ) = 370 - (33+3) = \bf{334}</math> values.
 +
 
 +
 
 +
Case <math>C</math>: <math>11|x</math>, but <math>3 \nmid x</math> and <math>101 \nmid x</math>.
 +
 
 +
Much like previous case, <math>abcd</math> is <math>11x</math>, so  <math>x \leq \frac{9999}{11} = 909  </math>, 
 +
 
 +
giving <math>\left\lfloor \frac{909}{11}\right\rfloor -  \left(\left\lfloor\frac{909}{11 \cdot 3}\right\rfloor + \left\lfloor\frac{909}{11 \cdot 101}\right\rfloor \right) = 82 - (27 + 0) = \bf{55}</math> values.
 +
 
 +
 
 +
Case <math>D</math>: <math>3|x</math> and <math>11|x</math> (so <math>33|x</math>), but <math>101 \nmid x</math>. 
 +
 
 +
Here, <math>abcd</math> is <math>99x</math>, so  <math>x \leq \frac{9999}{99} = 101  </math>,
 +
 
 +
giving <math>\left\lfloor \frac{101}{33}\right\rfloor - \left\lfloor \frac{101}{33 \cdot 101}\right\rfloor = 3-0 = \bf{3}</math> values.
 +
 
 +
 
 +
Case <math>E</math>: <math>101|x</math>.
 +
 
 +
Here, <math>abcd</math> is <math>101x</math>, so  <math>x \leq \frac{9999}{101} = 99  </math>,
 +
 
 +
giving <math>\left\lfloor \frac{99}{101}\right\rfloor = \bf{0}</math> values, so we don't need to account for multiples of <math>3</math> and <math>11</math>.
 +
 
 +
To sum up, the answer is <cmath>6000+334+55+3+0\equiv\boxed{392} \pmod{1000}.</cmath>
 +
 
 +
===Clarification===
 +
In this context, when the solution says, "Then <math>abcd=9x</math> to leave 3 uncancelled, and <math>x=3p</math>," it is a bit vague. The best way to clarify this is by this exact example - what is really meant is we need to divide by 9 first to achieve 1111, which has no multiple of 3; thus, given that the fraction x/y is the simplest form, x can be a multiple of 3.
 +
 
 +
Similar explanations can be said when the solution divides 9999 by 11, 101, and uses that divided result in the PIE calculation rather than 9999.
 +
 
 +
'''mathboy282'''
 +
 
 +
==Solution 2 (similar''(?)'' to solution 1 but reworded (not ''exactly'' for clarity))==
 +
 
 +
<cmath>\text{To begin, we notice that all repeating decimals of the form }0.\overline{abcd}\text{ where }a,b,c,d\text{ are digits can be expressed of the form }\frac{\overline{abcd}}{9999}\text{.}</cmath>
 +
<cmath>\text{However, when }\overline{abcd}\mid 9999\text{, the fraction is not in lowest terms.}</cmath>
 +
<cmath>\text{Since }9999 = 3^2 \cdot 11 \cdot 101\text{, } x\mid 9999\iff x\mid 3\lor x\mid 11\lor x\mid 101\text{.}</cmath>
 +
<cmath>\text{(For those of you who have no idea what that meant, it means every divisor of 9999 is a divisor of at least one of the following: )}</cmath>
 +
<cmath>(3)</cmath>
 +
<cmath>(11)</cmath>
 +
<cmath>(101)</cmath>
 +
<cmath>\text{(Also, I'm not going to give you explanations for the other logic equations.)}</cmath>
 +
<cmath>\text{Let's say that the fraction in lowest terms is }\frac{x}{y}\text{.}</cmath>
 +
<cmath>\text{If }x\mid 101\text{, then }99\mid y\text{ but that can't be, since }0\text{ is the only multiple of }101\text{ below }99\text{.}</cmath>
 +
<cmath>\exists! f(f\in\mathbb{N}\land f\neq 1\land\exists g(g\nleq 0 \land x \mid f^g))\implies f=3\lor f=11 (1)</cmath>
 +
<cmath>\text{If (1) is true, then we have two cases. If it isn't, we also have two cases.}</cmath>
 +
<cmath>\textbf{\textit{Case 1: }}f=3</cmath>
 +
<cmath>y=1111\land x=3z\implies 1\leq z\leq 370</cmath>
 +
<cmath>370-33-3^{[1]}=334</cmath>
 +
<cmath>\textbf{\textit{Case 2: }}f=11</cmath>
 +
<cmath>y=909\land x=11z\implies 1\leq z\leq 82</cmath>
 +
<cmath>82-27=55</cmath>
 +
<cmath>\textbf{\textit{Case 3: }}\neg\exists! f(f\in\mathbb{N}\land f\neq 1\land\exists g(g\nleq 0 \land x \mid f^g))\land \exists f(f\in\mathbb{N}\land f\neq 1\land\exists g(g\nleq 0 \land x \mid f^g))=</cmath>
 +
<cmath>\exists f_1(f_1\in\mathbb{N}\land f_1\neq 1\land\exists g_1(g_1\nleq 0 \land x \mid f_1^{g_1})\land\exists f_2(f_2\neq f_1\land f_2\in\mathbb{N}\land f_2\neq 1\land\exists g_2(g_2\nleq 0 \land x \mid f_2^{g_2}))\implies f_1=3\land f_2=11\lor f_1=11\land f_2=3</cmath>
 +
<cmath>y=101\land x=33z\implies 1\leq z\leq 3</cmath>
 +
<cmath>\textbf{\textit{Case 4: }}\neg\exists f(f\in\mathbb{N}\land f\neq 1\land\exists g(g\nleq 0 \land x \mid f^g))</cmath>
 +
<cmath>\Phi (9999)=6000</cmath>
 +
<cmath>\textbf{\textit{Grand Finale}}</cmath>
 +
<cmath>\text{Adding the outcomes, }N=6000+334+55+3=6392\equiv\boxed{392}\text{ (mod 1000).}</cmath>
 +
 
 +
<cmath>\textit{[1] This is to make sure that 3 is the \textbf{only} factor of x}</cmath>
 +
 
 +
===Note===
 +
 
 +
<cmath>\text{When I tried to write LaTeX, AoPS kept putting the LaTeX on a new line so I gave up and put most of it in LaTeX  instead.}</cmath>
 +
<cmath>\text{Some of the text in this section is just normal.}</cmath>
 +
<cmath>\text{Example:}</cmath>
 +
 
 +
Normal text <cmath>\text{This is some LaTeX.}</cmath> More normal text
 +
 
 +
<cmath>\text{If any of you can fix this issue, please do so.}</cmath>
 +
 
 +
~ [[User:Afly|Afly]] ([[User talk:Afly|talk]])
 +
 
 +
==Video Solution==
 +
https://youtu.be/0FZyjuIOHnA
 +
 
 +
https://MathProblemSolvingSkills.com
  
 
==See Also==
 
==See Also==

Latest revision as of 21:53, 2 January 2024

Problem

Let $S$ be the set of all rational numbers that can be expressed as a repeating decimal in the form $0.\overline{abcd},$ where at least one of the digits $a,$ $b,$ $c,$ or $d$ is nonzero. Let $N$ be the number of distinct numerators obtained when numbers in $S$ are written as fractions in lowest terms. For example, both $4$ and $410$ are counted among the distinct numerators for numbers in $S$ because $0.\overline{3636} = \frac{4}{11}$ and $0.\overline{1230} = \frac{410}{3333}.$ Find the remainder when $N$ is divided by $1000.$

Solution 1

$0.\overline{abcd}=\frac{abcd}{9999} = \frac{x}{y}$, $9999=9\times 11\times 101$.

Then we need to find the number of positive integers $x$ that (with one of more $y$ such that $y|9999$) can meet the requirement $1 \leq {x}\cdot\frac{9999}{y} \leq 9999$.

Make cases by factors of $x$. (A venn diagram of cases would be nice here.)


Case $A$:

$3 \nmid x$ and $11 \nmid x$ and $101 \nmid x$, aka $\gcd (9999, x)=1$.

Euler's totient function counts these: \[\varphi \left(3^2 \cdot 11 \cdot 101 \right) = ((3-1)\cdot 3)(11-1)(101-1)= \bf{6000}\] values (but it's enough to note that it's a multiple of 1000 and thus does not contribute to the final answer)

Note: You don't need to know this formula. The remaining cases essentially re-derive the same computation for other factors of $9999$. This case isn't actually different.

The remaining cases have $3$ (or $9$), $11$, and/or $101$ as factors of $abcd$, which cancel out part of $9999$. Note: Take care about when to use $3$ vs $9$.


Case $B$: $3|x$, but $11 \nmid x$ and $101 \nmid x$.

Then $abcd=9x$ to leave 3 uncancelled, and $x=3p$, so $x \leq \frac{9999}{9} = 1111$, giving:

$x \in 3 \cdot \{1, \dots \left\lfloor \frac{1111}{3}\right\rfloor\}$,

$x \notin (3\cdot 11) \cdot \{1 \dots \left\lfloor \frac{1111}{3\cdot 11}\right\rfloor\}$,

$x \notin (3 \cdot 101) \cdot \{1 \dots \left\lfloor \frac{1111}{3 \cdot 101}\right\rfloor\}$,

for a subtotal of $\left\lfloor \frac{1111}{3}\right\rfloor -  (\left\lfloor\frac{1111}{3 \cdot 11}\right\rfloor + \left\lfloor\frac{1111}{3 \cdot 101}\right\rfloor ) = 370 - (33+3) = \bf{334}$ values.


Case $C$: $11|x$, but $3 \nmid x$ and $101 \nmid x$.

Much like previous case, $abcd$ is $11x$, so $x \leq \frac{9999}{11} = 909$,

giving $\left\lfloor \frac{909}{11}\right\rfloor -  \left(\left\lfloor\frac{909}{11 \cdot 3}\right\rfloor + \left\lfloor\frac{909}{11 \cdot 101}\right\rfloor \right) = 82 - (27 + 0) = \bf{55}$ values.


Case $D$: $3|x$ and $11|x$ (so $33|x$), but $101 \nmid x$.

Here, $abcd$ is $99x$, so $x \leq \frac{9999}{99} = 101$,

giving $\left\lfloor \frac{101}{33}\right\rfloor - \left\lfloor \frac{101}{33 \cdot 101}\right\rfloor = 3-0 = \bf{3}$ values.


Case $E$: $101|x$.

Here, $abcd$ is $101x$, so $x \leq \frac{9999}{101} = 99$,

giving $\left\lfloor \frac{99}{101}\right\rfloor = \bf{0}$ values, so we don't need to account for multiples of $3$ and $11$.

To sum up, the answer is \[6000+334+55+3+0\equiv\boxed{392} \pmod{1000}.\]

Clarification

In this context, when the solution says, "Then $abcd=9x$ to leave 3 uncancelled, and $x=3p$," it is a bit vague. The best way to clarify this is by this exact example - what is really meant is we need to divide by 9 first to achieve 1111, which has no multiple of 3; thus, given that the fraction x/y is the simplest form, x can be a multiple of 3.

Similar explanations can be said when the solution divides 9999 by 11, 101, and uses that divided result in the PIE calculation rather than 9999.

mathboy282

Solution 2 (similar(?) to solution 1 but reworded (not exactly for clarity))

\[\text{To begin, we notice that all repeating decimals of the form }0.\overline{abcd}\text{ where }a,b,c,d\text{ are digits can be expressed of the form }\frac{\overline{abcd}}{9999}\text{.}\] \[\text{However, when }\overline{abcd}\mid 9999\text{, the fraction is not in lowest terms.}\] \[\text{Since }9999 = 3^2 \cdot 11 \cdot 101\text{, } x\mid 9999\iff x\mid 3\lor x\mid 11\lor x\mid 101\text{.}\] \[\text{(For those of you who have no idea what that meant, it means every divisor of 9999 is a divisor of at least one of the following: )}\] \[(3)\] \[(11)\] \[(101)\] \[\text{(Also, I'm not going to give you explanations for the other logic equations.)}\] \[\text{Let's say that the fraction in lowest terms is }\frac{x}{y}\text{.}\] \[\text{If }x\mid 101\text{, then }99\mid y\text{ but that can't be, since }0\text{ is the only multiple of }101\text{ below }99\text{.}\] \[\exists! f(f\in\mathbb{N}\land f\neq 1\land\exists g(g\nleq 0 \land x \mid f^g))\implies f=3\lor f=11 (1)\] \[\text{If (1) is true, then we have two cases. If it isn't, we also have two cases.}\] \[\textbf{\textit{Case 1: }}f=3\] \[y=1111\land x=3z\implies 1\leq z\leq 370\] \[370-33-3^{[1]}=334\] \[\textbf{\textit{Case 2: }}f=11\] \[y=909\land x=11z\implies 1\leq z\leq 82\] \[82-27=55\] \[\textbf{\textit{Case 3: }}\neg\exists! f(f\in\mathbb{N}\land f\neq 1\land\exists g(g\nleq 0 \land x \mid f^g))\land \exists f(f\in\mathbb{N}\land f\neq 1\land\exists g(g\nleq 0 \land x \mid f^g))=\] \[\exists f_1(f_1\in\mathbb{N}\land f_1\neq 1\land\exists g_1(g_1\nleq 0 \land x \mid f_1^{g_1})\land\exists f_2(f_2\neq f_1\land f_2\in\mathbb{N}\land f_2\neq 1\land\exists g_2(g_2\nleq 0 \land x \mid f_2^{g_2}))\implies f_1=3\land f_2=11\lor f_1=11\land f_2=3\] \[y=101\land x=33z\implies 1\leq z\leq 3\] \[\textbf{\textit{Case 4: }}\neg\exists f(f\in\mathbb{N}\land f\neq 1\land\exists g(g\nleq 0 \land x \mid f^g))\] \[\Phi (9999)=6000\] \[\textbf{\textit{Grand Finale}}\] \[\text{Adding the outcomes, }N=6000+334+55+3=6392\equiv\boxed{392}\text{ (mod 1000).}\]

\[\textit{[1] This is to make sure that 3 is the \textbf{only} factor of x}\]

Note

\[\text{When I tried to write LaTeX, AoPS kept putting the LaTeX on a new line so I gave up and put most of it in LaTeX  instead.}\] \[\text{Some of the text in this section is just normal.}\] \[\text{Example:}\]

Normal text \[\text{This is some LaTeX.}\] More normal text

\[\text{If any of you can fix this issue, please do so.}\]

~ Afly (talk)

Video Solution

https://youtu.be/0FZyjuIOHnA

https://MathProblemSolvingSkills.com

See Also

2022 AIME I (ProblemsAnswer KeyResources)
Preceded by
Problem 12
Followed by
Problem 14
1 2 3 4 5 6 7 8 9 10 11 12 13 14 15
All AIME Problems and Solutions

The problems on this page are copyrighted by the Mathematical Association of America's American Mathematics Competitions. AMC logo.png